184

Foro de discusion Sobre RFH

Moderador: Alberto

Responder
Derwyd
Cl
Cl
Mensajes: 167
Registrado: 29 Jun 2009, 16:50

184

Mensaje por Derwyd »

Dan por válida la 4, pero creo que la 2 también es correcta. Cuanto mayor es el índice de refracción, menor es la energía umbral para producir luz, y por tanto mas fotones de generarán.
Sheldon es Dios y yo su profeta.
Avatar de Usuario
Incógnita
I
I
Mensajes: 521
Registrado: 29 Ago 2008, 19:34

Re: 184

Mensaje por Incógnita »

Yo esta no la contesté, pero luego repasando también pensé que fuera la 2. Puede ser anulable.
Derwyd
Cl
Cl
Mensajes: 167
Registrado: 29 Jun 2009, 16:50

Re: 184

Mensaje por Derwyd »

Pues va a ser que no. Acabo de encontrar una fórmula para el número de fotones emitidos y depende inversamente del cuadrado del índice de refracción.
Sheldon es Dios y yo su profeta.
Avatar de Usuario
pacotem
Ca
Ca
Mensajes: 196
Registrado: 12 Feb 2008, 14:24

Re: 184

Mensaje por pacotem »

Hola Derwyd
Con esa fórmula la respuesta correcta sería la 5? ¿Dónde encontramos bibliografía?
Avatar de Usuario
pacotem
Ca
Ca
Mensajes: 196
Registrado: 12 Feb 2008, 14:24

Re: 184

Mensaje por pacotem »

Hola otra vez,
He encontrado esto en el google books, el problema es que no lo puedo bajar. No lo consigo con el google books.

Física Nuclear
Burcham
pag 119
Derwyd
Cl
Cl
Mensajes: 167
Registrado: 29 Jun 2009, 16:50

Re: 184

Mensaje por Derwyd »

Se puede impugnar. He subido al ftp la página del Burcham donde se ve la fórmula del número de fotones emitido y que por un lado, da por válida la afirmación de que para velocidades próximas a c no depende de la velocidad de la partícula (beta tiende a 1) y por otro que depende de 1/lambda. Por lo tanto, dos válidas, anulable.
Sheldon es Dios y yo su profeta.
Avatar de Usuario
pacotem
Ca
Ca
Mensajes: 196
Registrado: 12 Feb 2008, 14:24

Re: 184

Mensaje por pacotem »

Ok ,muchas gracias
Derwyd
Cl
Cl
Mensajes: 167
Registrado: 29 Jun 2009, 16:50

Re: 184

Mensaje por Derwyd »

Ahora me ha entrado la duda. El Knoll dice que la producción de fotones depende de 1/lambda^2. A ver si encuentro algo más.
Sheldon es Dios y yo su profeta.
noelia86
Si
Si
Mensajes: 137
Registrado: 19 Ene 2010, 22:54

Re: 184

Mensaje por noelia86 »

Entoncs, cual es la correcta???
Derwyd
Cl
Cl
Mensajes: 167
Registrado: 29 Jun 2009, 16:50

Re: 184

Mensaje por Derwyd »

Bueno, creo que ya lo tengo claro y no hay nada que rascar. El tema es que nos piden N(lambda) por intervalo de de longitudes de onda diferencial de lambda. En este caso la dependencia es 1/lambda^2.

\(\frac{dN}{dx}=\frac{2\pi z^2 \alpha}{\lambda^2}(1-\frac{1}{\beta^2 n(\lambda)})d\lambda\)

El 1/lambda del Burcham sale de integrar en las longitudes de onda, pero no es lo que piden.

EDITO: Lo subo si queréis, pero no hay nada que impugnar.
Sheldon es Dios y yo su profeta.
pasanvin
B
Mensajes: 45
Registrado: 20 Ago 2009, 14:45

Re: 184

Mensaje por pasanvin »

Según lo que he visto en el ftp, a mi me parece que la más correcta es la 2, así que voy a impugnarla
Derwyd
Cl
Cl
Mensajes: 167
Registrado: 29 Jun 2009, 16:50

Re: 184

Mensaje por Derwyd »

¿La 2? Si en la expresión que viene en el Burcham que subí al ftp para el número de fotones hay un n^2 en el denominador.
Sheldon es Dios y yo su profeta.
pasanvin
B
Mensajes: 45
Registrado: 20 Ago 2009, 14:45

Re: 184

Mensaje por pasanvin »

Miralo bien, el factor és

\(\left( 1-\frac{1}{\beta^2 n^2}\right)\)

con lo cual, si n augmenta, este factor disminuye, ademas he encontrado una tabla de resultados experimentales en otro libro que corrobora este resultado
Derwyd
Cl
Cl
Mensajes: 167
Registrado: 29 Jun 2009, 16:50

Re: 184

Mensaje por Derwyd »

Coño tienes razón, me despiste totalmente. Voy a subir la otra referencia que comentaba. ¿Puedes hacer lo mismo con la que tu dices?
Sheldon es Dios y yo su profeta.
pasanvin
B
Mensajes: 45
Registrado: 20 Ago 2009, 14:45

Re: 184

Mensaje por pasanvin »

Ahora después lo subo
Responder